Difference between revisions of "2021 AMC 10B Problems/Problem 4"

(Removed all unnecessary contents. I merged all contents on the corresponding AMC 12 page.)
(Tag: Replaced)
 
(12 intermediate revisions by 7 users not shown)
Line 1: Line 1:
At a math contest, <math>57</math> students are wearing blue shirts, and another <math>75</math> students are wearing yellow shirts. The 132 students are assigned into <math>66</math> pairs. In exactly <math>23</math> of these pairs, both students are wearing blue shirts. In how many pairs are both students wearing yellow shirts?
+
#REDIRECT [[2021_AMC_12B_Problems/Problem_2]]
 
 
<math>\textbf{(A)} ~23 \qquad\textbf{(B)} ~32 \qquad\textbf{(C)} ~37 \qquad\textbf{(D)} ~41 \qquad\textbf{(E)} ~64</math>
 

Latest revision as of 05:18, 4 March 2021